\documentclass[a5paper,10pt]{article} \usepackage{myXsim} \usepackage{tasks} % Title Page \title{DM2 \hfill ZENAGUI Yanis} \tribe{TST} \date{\hfillÀ render pour le Mercredi 24 février} \xsimsetup{ solution/print = false } \begin{document} \maketitle \begin{exercise}[subtitle={Loi binomiale}] Trois personnes s'apprêtent à passer le portique de sécurité. On suppose que pour chaque personne la probabilité que le portique sonne est égale à $0.23$. Soit $X$ la variable aléatoire donnant le nombre de personnes faisant sonner le portique, parmi les 3 personnes de ce groupe. \begin{enumerate} \item Tracer l'arbre représentant le situation. \item Justifier que $X$ suit une loi binomiale dont on précisera les paramètres. \item Quelle est la probabilité qu'une seule personne fasse sonner le portique? \item Calculer puis interpréter les probabilités suivantes \[ P(X = 0) \qquad \qquad P(X \geq 2) \] \item Calculer l'espérance de $X$ et interpréter le résultat. \end{enumerate} \end{exercise} \begin{solution} \begin{enumerate} \item \begin{tikzpicture}[sloped] \node {.} child {node {$0$} child {node {$0$} child {node {$0$} edge from parent node[above] {0.77} } child {node {$1$} edge from parent node[above] {0.23} } edge from parent node[above] {0.77} } child[missing] {} child {node {$1$} child {node {$0$} edge from parent node[above] {0.77} } child {node {$1$} edge from parent node[above] {0.23} } edge from parent node[above] {0.77} } edge from parent node[above] {0.77} } child[missing] {} child[missing] {} child[missing] {} child { node {$1$} child {node {$0$} child {node {$0$} edge from parent node[above] {0.77} } child {node {$1$} edge from parent node[above] {0.23} } edge from parent node[above] {0.77} } child[missing] {} child {node {$1$} child {node {$0$} edge from parent node[above] {0.77} } child {node {$1$} edge from parent node[above] {0.23} } edge from parent node[above] {0.77} } edge from parent node[above] {0.23} } ; \end{tikzpicture} \item Chaque personne a 2 possibilités (1: fait sonner ou 2: ne fait pas sonner) et l'on fait passer 3 personnes ce qui correspond à une répétition identique et aléatoire. On peut donc modéliser la situation par une loi binomiale. \[ X \sim \mathcal{B}(3; 0.76) \] \item Probabilité qu'une seule personne fasse sonner le portique. On voit qu'il y a 3 branches qui correspondent à cette situation dont \[ P(X = 1) = 3 \times 0.23^1 \times 0.77^2 \approx 0.409 \] \item \[ P(X = 0) = 0.77^3 \approx 0.457 \] \[ P(X \geq 2) = P(X = 2) + P(X = 3) = 3 \times 0.23^2 \times 0.77^1 + 0.23^3 \approx 0.134 \] \item Il faut d'abord tracer le tableau résumant la loi de probabilité: \begin{center} \begin{tabular}{|c|*{4}{c|}} \hline Valeur & 0 & 1 & 2 & 3 \\ \hline Probabilité & $0.457$ & $0.409$ & $0.122$ &$0.012$ \\ \hline \end{tabular} \end{center} On peut alors calculer l'espérance \[ E[X] = 0 \times 0.457 + 1 \times 0.409 + 2 \times 0.122 + 3 \times 0.012 = 0.69 \] On peut donc estimer qu'il y aura en moyenne $0.69$ personnes qui feront sonner le portique sur les 3 personnes. \end{enumerate} \end{solution} \begin{exercise}[subtitle={Équation puissance}] Résoudre les équations et inéquations suivantes \begin{multicols}{2} \begin{enumerate} \item $10^x = 15$ \item $7^x = 38$ \item $0.66^x \leq 14$ \item $9 \times 0.36^x = 21$ \end{enumerate} \end{multicols} \end{exercise} \begin{solution} Les solutions ci-dessous ne sont pas justifiée car l'ordinateur ne sait pas faire. Par contre, vous vous devez savoir justifier vos réponses! \begin{enumerate} \item $x = \log(15)$ \item $x = \frac{\log(38)}{\log(7)}$ \item Il faut faire attention quand on divise par un log car ce dernier peut être négatif ce qui est le cas ici. Il faut donc pense à changer le sens de l'inégalité. $x \geq \frac{\log(14)}{\log(0.66)}$ \item Il faut penser à faire la division à par $9$ avant d'utiliser le log car sinon, on ne peut pas utiliser la formule $\log(a^n) = n\times \log(a)$. $x = \frac{\log(2.33)}{\log(0.36)}$ \end{enumerate} \end{solution} \begin{exercise}[subtitle={Étude de fonctions}] Soit $f(x) = - 5x^3 + 375x^2 - 4515x + 9$ une fonction définie sur $\R$. \begin{enumerate} \item Calculer $f'(x)$ la dérivée de $f(x)$. \item Calculer $f'(43)$ et $f'(7)$. \item En déduire une forme factorisée de $f'(x)$. \item Étudier le signe de $f'(x)$ et en déduire les variations de $f(x)$. \item Est-ce que la fonction $f(x)$ admet un maximum ou un minimum? Si oui, calculer sa valeur. \end{enumerate} \end{exercise} \begin{solution} \begin{enumerate} \item Dérivée de $f(x)$: $f'(x) = - 15x^2 + 750x - 4515$ \item \begin{align*} f'(43) &= - 15 \times 43^{2} + 750 \times 43 - 4515\\&= - 15 \times 1849 + 32250 - 4515\\&= - 27735 + 27735\\&= 0 \end{align*} \begin{align*} f'(7) &= - 15 \times 7^{2} + 750 \times 7 - 4515\\&= - 15 \times 49 + 5250 - 4515\\&= - 735 + 735\\&= 0 \end{align*} Donc $x = 43$ et $x=7$ sont des racines de $f'(x) = - 15x^2 + 750x - 4515$. \item On en déduit la forme factorisée suivante \[ f'(x) = -15 (x - 43)(x-7) \] \item Pas de correction disponible \item À causes des branches extérieurs, la fonction $f(x)$ n'a pas de maximum ou de minimum. \end{enumerate} \end{solution} %\printsolutionstype{exercise} \end{document} %%% Local Variables: %%% mode: latex %%% TeX-master: "master" %%% End: